Esercizi misti limiti 8

Esercizi misti sui Limiti

Home » Esercizi misti limiti 8

Richiamiamo di seguito solo i principali risultati che verranno utilizzati per la risoluzione degli esercizi. Per i richiami teorici più completi si rimanda alle dispense di teoria sui limiti notevoli , alla dispensa sui simboli di Landau e a quella sulle forme indeterminate.

Teorema 1. 

Siano f, g\colon A \subseteq \mathbb{R}\to \mathbb{R}, sia x_0 \in \mathbb{R} \cup \{\pm \infty\} un punto di accumulazione per A. Si assuma che

    \[\exists \lim\limits_{x \to x_0} f(x) =: \ell_1, \qquad \exists \lim\limits_{x \to x_0} g(x) =: \ell_2,\]

allora, ogni qualvolta l’espressione a destra non è un forma indeterminata, si ha:

    \[\begin{aligned} \exists \; \lim\limits_{x \to x_0}(f\pm g)(x) & =\ell_1 \pm \ell_2 \\ \exists \; \lim\limits_{x \to x_0}(f\cdot g)(x) & = \ell_1 \cdot \ell_2, \end{aligned}\]

Se x_0 è un punto di accumulazione per \{x \in A \colon g(x) \neq 0\}, allora si ha:

    \[\exists \; \lim\limits_{x \to x_0} \left( \dfrac{f}{g}\right)(x) = \dfrac{\ell_1}{\ell_2},\]

ogni qualvolta l’espressione a destra esiste e non è una forma indeterminata.

 

Teorema 2 – Teorema di sostituzione. 

Sia f\colon A \subseteq \mathbb{R}\to \mathbb{R} e sia x_0 \in \mathbb{R}\cup \{\pm \infty\}. Si assuma che

    \[\exists \lim\limits_{x \to x_0} f(x) = \ell \in \mathbb{R}\cup \{\pm \infty\}.\]

Sia I(\ell) un intorno di \ell e sia g \colon I(\ell) \to \mathbb{R} tale che

  1. se \ell \in \mathbb{R}, g è continua in \ell;
  2. se \ell = \pm \infty, allora esiste \lim\limits_{y \to \ell}g(y).

Allora,

    \[\lim\limits_{x \to x_0} g(f(x)) = \lim\limits_{y \to \ell}g(y).\]

 

 

Teorema 3 – Teorema di L’Hôpital. 

Siano A\subset \mathbb{R} e x_0\in \mathbb{R}\cup \{\pm\infty\} punto di accumulazione per A. Siano f,g:A\setminus \{x_0\}\rightarrow \mathbb{R} derivabili nel loro dominio e inoltre si supponga g^\prime(x)\neq0 \, \, \forall x \in I \setminus \{x_0\} . Se f,g sono entrambe infinitesime o infinite per x \rightarrow x_0 e se esiste il seguente limite

    \[\lim_{x \rightarrow x_0}\dfrac{f^\prime(x)}{g^\prime(x)}=\ell \in \mathbb{R}\cup \{\pm\infty\},\]

allora

    \[\lim_{x \rightarrow x_0}\dfrac{f(x)}{g(x)}=\ell.\]

 

 

Testi degli esercizi

Esercizio 8  (\bigstar\bigstar\bigstar\largewhitestar\largewhitestar). Calcolare, se esistono, i seguenti limiti:

    \[\begin{aligned} &1.\quad  \lim_{x\to0}\left(2e^x-e^{2x}\right)^{1/x^2};\\[10pt] & 2.\quad  \lim_{x\to0^+} (x+\cos x)^{\frac{1}{x}};\\[10pt] &3.\quad \lim_{x\to0^+}\dfrac{\arccos(1-x^2)}{x};\\[10pt] &4.\quad \lim_{x\to a}\left(2-\dfrac{x}{a} \right)^{\displaystyle\tan\left(\frac{\pi x}{2a} \right)}, \qquad a \in \mathbb{R}\setminus\{0\};\\[10pt] &5.\quad \lim_{x\to 0^+} \dfrac{(\sin x)^x-1}{x^x-1}.\end{aligned}\]

 
Svolgimento.
 
1. Manipolando il limite dato si ha:

    \[\begin{aligned} \lim_{x\to0}\left(2e^x-e^{2x}\right)^{1/x^2} = & \lim_{x\to0}\exp\frac{\ln\left(2e^x-e^{2x}\right)}{x^2}\\ =& \exp\lim_{x\to0}\frac{\ln\left(2e^x-e^{2x}\right)}{x^2}\\ = & \exp\lim_{x\to0}\frac{\ln\left[1-(1-e^x)^2\right]}{x^2} \\ = & \exp\lim_{x\to0}\left[\frac{-(1-e^x)^2}{x^2}\frac{\ln\left[1-(1-e^x)^2\right]}{-(1-e^x)^2}\right] \\ = & \exp\lim_{x\to0}\frac{-(1-e^x)^2}{x^2} = \exp\lim_{x\to0} \left[-\left(\frac{e^x-1}{x} \right)^2 \right]\\ = & \exp(-1) = \frac{1}{e}. \end{aligned}\]

 
2. Applicando i limiti notevoli si ottiene:

    \[\begin{aligned} \lim_{x\to0^+} (x+\cos x)^{\frac{1}{x}}=&\lim_{x\to0^+} \left(\left(1+\left(x+\cos x-1\right)\right)^{\frac{1}{x+\cos x -1}}\right)^{\frac{x+\cos x-1}{x}}\\ =&\lim_{x\to0^+} \left(\left(1+\left(x+\cos x-1\right)\right)^{\frac{1}{x+\cos x -1}}\right)^{1-\left(\frac{1-\cos x}{x^2}\right)x}=e. \end{aligned}\]

 
3. Poiché il limite si presenta come una forma indeterminata [\infty/\infty], applicando il teorema 3 si ha:

    \[\begin{aligned} \lim_{x\to0^+}\dfrac{\arccos(1-x^2)}{x}=&\lim_{x\to0^+}\dfrac{2x}{\sqrt{1-(1-x^2)^2}}\\ =&\lim_{x\to0^+}\dfrac{2x}{\sqrt{1-1-x^4+2x^2}}\\ =& \lim_{x\to0^+} \dfrac{2x}{x\sqrt{-x^2+2}}=\sqrt{2}. \end{aligned}\]

 
4. Effettuando la sostituzione y = x-a, in virtù del teorema 2, si ha:

    \[\begin{aligned} \lim_{x\to a}\left(2-\dfrac{x}{a} \right)^{\displaystyle\tan\left(\frac{\pi x}{2a} \right)}&=\lim_{y\to 0}\left(2-\dfrac{y+a}{a}\right)^{\tan\left(\dfrac{\pi\left(y+a\right)}{2a}\right)}\\ &\overset{\clubsuit}{=}\lim_{y\to 0}\left(1-\dfrac{y}{a}\right)^{-\cot\left(\dfrac{\pi y}{2a}\right)}\\ &= \lim_{y\to 0}\left(\left(1-\dfrac{y}{a}\right)^{-\dfrac{a}{y}}\right)^{\left(\dfrac{\dfrac{\pi y}{2a}}{\tan\left(\dfrac{\pi y}{2a}\right)}\right)\cdot\left(\dfrac{y}{a}\cdot\dfrac{1}{\dfrac{\pi y}{2a}}\right)}=e^{\frac{2}{\pi}}. \end{aligned}\]

dove nel passaggio \clubsuit abbiamo sfruttato

    \[\tan\left(\dfrac{\pi y}{2a} + \dfrac{\pi}{2} \right) = - \cot\left( \dfrac{\pi y}{2a}\right).\]

 
5. Manipolando l’espressione del limite dato si ha:

    \[\begin{aligned} \lim_{x\to 0^+} \dfrac{(\sin x)^x-1}{x^x-1}=&\lim_{x\to 0^+} \dfrac{e^{x\ln\sin x}-1}{x^x-1}\\ =& \lim_{x\to 0^+}\left(\dfrac{e^{x\ln\sin x}-1}{x\ln\sin x}\right)\cdot\left(\dfrac{x\ln x}{x^x-1}\right)\cdot\left(\dfrac{\ln\sin x }{\ln x}\right). \end{aligned}\]

Osserviamo che

    \[\lim_{x\to 0^+}x\ln \sin x =\lim_{x\to 0^+}\dfrac{\ln\sin x }{x^{-1}}\overset{\clubsuit}{=}\lim_{x\to 0^+}\dfrac{\cos x }{-x^{-2}\sin x }=\lim_{x\to 0^+}-x\cos x \left(\dfrac{x}{\sin x}\right)\overset{\star}{=}0\]

e

    \[\lim_{x\to 0^+}x\ln x=	\lim_{x\to 0^+}\frac{\ln x}{x^{-1}}\overset{\clubsuit}{=}\lim_{x\to 0^+}\frac{x^{-1}}{-x^{-2}}=\lim_{x\to 0^+}-x=0,\]

dove in \clubsuit abbiamo applicato il teorema 3. Segue dunque che per il primo ed il terzo limite possiamo applicare il limite notevole \lim_{x\to 0}\dfrac{e^{f(x)}-1}{f(x)}=1, con f(x) infinitesima per x\to0, da cui si ha:

    \[\begin{aligned} \lim_{x\to 0^+} \dfrac{(\sin x)^x-1}{x^x-1}=& \lim_{x\to 0^+}\left(\dfrac{e^{x\ln\sin x}-1}{x\ln\sin x}\right)\cdot\left(\dfrac{x\ln x}{x^x-1}\right)\cdot\left(\dfrac{\ln\sin x }{\ln x}\right)=1. \end{aligned}\]

error: Il contenuto è protetto!!